Questions tagged [analytic-number-theory]

A beautiful blending of real/complex analysis with number theory. The study involves distribution of prime numbers and other problems and helps giving asymptotic estimates to these.

Filter by
Sorted by
Tagged with
0 votes
0 answers
26 views

Are the coefficients in the stationary phase approximation computed explicitly somewhere

In Stein's "Harmonic analysis" book, page 334, one can find the asymptotic expansion An instructive proof is given for the case $k=2$. It is clear enough to generalize to the cases $k\geq ...
Medo's user avatar
  • 676
5 votes
2 answers
213 views

Prime differences and zero multiplicity

Paul Erdős conjectured, for consecutive primes, that: $$\sum_{p_n < x} (p_n -p_{n-1})^2 = O(x \log x)$$ Let's call this hypothesis EH. Assuming the Riemann hypothesis (RH), Selberg showed that a ...
Felixson's user avatar
  • 162
2 votes
0 answers
96 views

Gaussian primes in translations of lattices in $\mathbb{Z}[i]$

I am considering undertaking some independent research in my summer break studying Gaussian primes in translations of lattices in $\mathbb{Z}[i]$, i.e. sets of the form $ \{ a+sx+tw:s,t \in \mathbb{Z} ...
Daniel Lang's user avatar
1 vote
0 answers
57 views

Tuples of natural numbers with no mutual divisibility and large reciprocal sums

Standard apology in case this is something simple, as I'm not a number theorist. Let $E_1, \dots, E_n$ be disjoint finite sets of natural numbers, such that for any $a_1 \in E_1, \dots, a_n \in E_n$, ...
Sophie M's user avatar
  • 675
3 votes
0 answers
332 views

Analytic number theory and condensed mathematics

As of 2024, are there current or planned applications of condensed mathematics to analytic number theory? If so, what are suggested readings?
Jon23's user avatar
  • 127
1 vote
0 answers
50 views

How to check that a number probably/likely has a divisor having a specific bit length/in range?

Given a randomly generated $\alpha\in\Bbb N$ where $\alpha$ is large thus hard to factor (no small prime composites). How to check that a divisor $F\in\Bbb N$ with a specific bitlengh $n\in\Bbb N∧n<...
user2284570's user avatar
1 vote
0 answers
51 views

A question on generalized bases

I just came to know that it is possible to define a generalized base as an infinite sequence of natural numbers $\mathbf b=(b_1,b_2,\dots)$ where $b_i\ge 2$ for all $i$. With this definition, any $m\...
Dumbest person on earth's user avatar
5 votes
1 answer
350 views

Lindelöf hypotheses for derivatives of zeta

The Lindelöf hypothesis says that if we have: $$\zeta(\sigma+iT)=\mathcal O(T^a)$$ Then if one considers $\sigma=1/2$ then $\inf a=0$. Further, from convexity and the functional equation this implies ...
psubodiosa's user avatar
0 votes
0 answers
52 views

Average of number of divisors of shifted exponential sequence

Let $a$ be a fixed positive integer greater than 1. We define the sequence $u_n=a^{n}-1$ for all positive integers $n$. Then are there any results in literature for asymptotic value of the sum $$\sum_{...
Hhhhhhhhhhh's user avatar
  • 1,032
2 votes
0 answers
162 views

Asymptotics of $\vartheta(x+y)-\vartheta(x)$, where $\vartheta$ is the Chebyshev function, when $y\in[x^\alpha, x]$ for some $\alpha\in(0,1)$

Introduction Consider $\vartheta$ to be the Chebyshev function, that is, $\vartheta(x)$ denotes, for $x\in\mathbb N$, the sum $\sum_{p\le x,\ p\text{ prime}} \ln p$. I am interested in asymptotics for ...
Maximilian Janisch's user avatar
2 votes
1 answer
130 views

Upper bound of a product of sines

Consider the function $$ f_n(t)= \prod_{1 \leq k \leq n-1,\\ \gcd(k,n)=1} \sin\Big(t-\frac{k \pi}{n}\Big),\quad t \in [0,\pi].$$ I wonder whether it is possible to compute some nontrivial upper ...
AgnostMystic's user avatar
4 votes
0 answers
157 views

Effective bound for odd numbers expressed as sums of three primes

I am interested in the representation of odd numbers greater than five as sums of three primes, inspired by Harald Helfgott's seminal proof of the ternary Goldbach conjecture and the nuanced findings ...
Anton Rechenauer's user avatar
6 votes
0 answers
564 views

Generating functions in countable commutative monoids

Let $f: \mathbb{N}_0 \rightarrow \mathbb{C}$ be a function. The power series of $f$ can be viewed as the function $\mathscr{P}_f : q \mapsto \sum_{n \in \mathbb{N}_0}^{} f(n)q^n$ where $q \in \mathbb{...
Tian Vlašić's user avatar
0 votes
0 answers
158 views

Sum of reciprocals of primes dividing Mersenne numbers

Let $p$ be a prime. Then it has been proven by Erdős that $$\sum_{p\mid 2^{n}-1}\frac1{p} \ll \log \log \log n.$$ Let $c<1$ be a positive constant. Then can we prove that $$\sum_{p\mid 2^{n}-1}\...
Hhhhhhhhhhh's user avatar
  • 1,032
2 votes
0 answers
114 views

Techniques of showing the Order

In their paper The integral of Riemann Xi Function, Lagarias and Montague show that the integral $$\Xi_{\lambda}^{-1}(z)=2\int_0^{\infty}e^{\lambda u^2}\phi(u)\Big( \frac{\sin zu}{u}\Big)\;du$$ is ...
Tokita Ohma's user avatar
0 votes
1 answer
285 views

A basic conjecture/observation on the Riemann $\xi$-function

Based on computations I have made the following mini-conjecture: For any zeta zero $s_0$ with $|s_0|\geq|1-s_0|$ and for $0<\tau<1$ define $M_\tau=|s_0|(1+(1-\tau)^2)$. Let $\xi$ stand for the ...
Alexis's user avatar
  • 31
3 votes
1 answer
666 views

Infinite dimensional lattice for integers and the Riemann hypothesis?

It is known that for each finite set of primes $p$ we have: $\log(p)$ are linear independent over the rational numbers. We have $\log(ab) = \log(a)+\log(b)$ and $\log(n) = \sum_{p |n}v_p(n) \log(p)$. ...
mathoverflowUser's user avatar
1 vote
1 answer
176 views

Classification of L functions and Dirichlet series by poles

I am interested in the connection between particular Dirichlet series' abscissa of convergence and the poles of L-functions. Let $D(z) = \sum_{n=1}^\infty\frac{a_n}{n^z}$ be a Dirichlet series ...
asdfasdgasdfgasfgasdf's user avatar
3 votes
2 answers
249 views

Equidistribution on $\mathrm{SU}_2$

Let $F_{a_1,a_2}$ be the free group with a free generating set $\{a_1,a_2\}$ of two elements, and for any $n\in\mathbb{N}$, set $A_n=\{\text{reduced words in } F_{a_1,a_2} \text{with length} \leqslant ...
Local's user avatar
  • 86
2 votes
1 answer
163 views

Modified Gauss Sum when the characters have different period

Let $\chi$ be a Dirichlet character mod q, and \begin{eqnarray} t(\chi)=\sum_{n=1}^{q}\chi(n)e(\frac{n}{2q}). \end{eqnarray} Do we have a bound or formula for $t(\chi)$ similar to that of the usual ...
Jonathan Lam's user avatar
3 votes
1 answer
179 views

Generators of the ideal class group

Theorem 4 of Eric Bach's "Explicit bounds for primality testing and related problems" states the following: Let $K$ be a number field of degree greater than 1. Let $d$ be the absolute value ...
Rashad Ek's user avatar
3 votes
0 answers
92 views

Error function of the second moment of the divisor function

It is easy to show that the second moment of the divisor function has asymptotics: $$\sum_{n\leq x} d_0(n)^2 = xP(\log(x))+E_2(x)$$ Where $P$ is some polynomial and that: $$E_2 = o(x)$$ Previously, ...
psubodiosa's user avatar
2 votes
0 answers
148 views

Electrostatic potential energy of point-charges at primes up to $x$

Given a positive real (or integral) number $x$ we consider the electrostatic potential energy of equal point charges at all primes up to $x$ given by $$E(x)=\sum_{p_1<p_2\leq x}\frac{1}{p_2-p_1}$$ ...
Roland Bacher's user avatar
5 votes
0 answers
162 views

Empirical bounds on $\left|\frac{\zeta'(1+it)}{\zeta(1+it)}\right|$

It is reasonable to expect that $$\left|\frac{\zeta'(1+it)}{\zeta(1+it)}\right| < 2 \log \log t$$ for all $t\geq 4$ (say): a somewhat stronger bound is known for $t\geq 10^{165}$ or so (Theorem 5 ...
H A Helfgott's user avatar
  • 19.3k
1 vote
1 answer
147 views

Density of a set of numbers whose prime factors are defined by congruences

Let $S$ be the set of positive integers not divisible by $3$ where if $p$ is a prime factor of $n \in S$ and $p \equiv 1\bmod 3$ then $p^2$ does not divide $n$, but if $p\equiv2 \bmod 3$ then $p^2$ ...
AsksQuestionsAboutMath's user avatar
3 votes
0 answers
209 views

Number of zeros of the zeta function along horizontal lines

Are there any known results about the number of zeros of the zeta function along horizontal lines of the complex plane? The Riemann hypothesis states that for any such line the number is at most 1, ...
Alexis's user avatar
  • 31
4 votes
2 answers
422 views

Reference request - Pillai-Selberg Theorem

I want to find a proof of the following claim. Let $\Omega(n)$ denote the number of prime factors of an integer $n$ counted with multiplicity. Then $\Omega(n)$ equidistributes over residue classes. ...
alidixon222's user avatar
2 votes
1 answer
362 views

Limit of an infinite series with quadratic arguments

I have encountered a limiting process on some infinite series. So, I would like to ask: QUESTION. Assume $n$ is an even positive integer. Is this true? $$\lim_{r\rightarrow1^{-}}\sum_{j=1}^{\infty}\...
T. Amdeberhan's user avatar
0 votes
1 answer
124 views

A question about the setup of zero density estimates for Dirichlet $L$-functions

For $L(s,\chi)= \sum_{n \geq 1}\frac{\chi(n)}{n^s}$, where $s = \sigma + it$, we define the function $N(\sigma, T, \chi)$ which counts the zeros $\rho = \beta + i\gamma$ for which $L(\rho, \chi) =0$ ...
Josh's user avatar
  • 1
4 votes
0 answers
438 views

There are infinitely many prime which have arbitrary large gap in their digits in particular base expansion

Consider $m$ and $r$ is any fixed positive integer and $t$ is a variable $(t=0,1,2,3,...)$. Below, $[a]$ denotes the greatest integer function of $a$ (or floor function). Claim 1 : There exists a ...
Pruthviraj's user avatar
1 vote
1 answer
435 views

Conjectured upper bound on the maximum value of the absolute value of the Möbius function in the poset of multiplicative partitions under refinement

PRELIMINARIES: Consider the poset $(\mathcal{P}_n, \leq_r)$ of the (unordered) multiplicative partitions of $n$ partially ordered under refinement (for all $\lambda, \lambda’ \in \mathcal{P}_n$, we ...
Tian Vlašić's user avatar
6 votes
2 answers
636 views

On the asymptotic behaviour of the series $\sum_{n=1}^{\infty} \left ( \frac{\zeta(ns)}{n}+\frac{s}{1-ns}\right )$ near $s=0$

I am interested in determining the behaviour of the the series/function $$f(s)=\sum_{n=1}^{\infty} \left ( \frac{\zeta(ns)}{n}+\frac{s}{1-ns}\right )$$ near $s=0$. It is clear that $f(0)$ is undefined....
Tian Vlašić's user avatar
3 votes
0 answers
103 views

Is there any notion of Poincaré series for Hermitian modular forms?

I have been studying modular forms and their generalisations for a year or so. It is a very interesting fact that the space of cusp forms $S_k$ is generated by the Poincaré series of exponential type (...
Ancient Antagonist's user avatar
1 vote
0 answers
180 views

Asymptotic behaviour of a sum involving Möbius function

(This is a cross-posted simplification of this question posted in MSE which did not have a complete answer.) I am trying to get the asymptotic behaviour when $n$ grows to infinity of a partial sum of ...
Juan Moreno's user avatar
1 vote
1 answer
155 views

prime distribution lower bound: Does the inequality hold for $s=19/10$?

Define a transformation of the prime counting function, $\pi(k)$, by $$J: (0,1) \longrightarrow (0,1) $$ where $$J(x)= \lim_{r \to \infty} \frac{\int_2^{rx} \pi(k) \, dk }{\int_2^r \pi(k) \, dk}$$ ...
53Demonslayer's user avatar
1 vote
1 answer
288 views

Is there any way to estimate this functions: $f(n)=\sum_{d|n}d\varphi(d)$ and $g(n)=\sum_{d|n}\frac{\varphi(d)}{d}$?

Let that $n$ be a natural number and $\varphi(n)$ be the Euler totient function. Is there any formula or estimation for computing functions $f,g$ such that: $$ f(n)=\sum_{d\mid n}d\varphi(d) $$ and $$ ...
Jamal Farokhi's user avatar
6 votes
1 answer
190 views

A strictly increasing, analytic function that goes through key points of the iterated logarithm?

Is it possible to create a function $f(x)$ that: is strictly increasing (at least for $x>0$) is real analytic goes through all the points where the iterated logarithm would increment value? i.e. [...
user5399200's user avatar
0 votes
0 answers
58 views

What is the lattice point distribution over binary quadratic forms?

Let $f(x,y)=x^2+ny^2$ be the binary quadratic form of interest and consider the lattice points $S=\{ (x,y,f(x,y)) \in \mathbb{N}^3 \}$. For simplicity, we keep things only on quadrant I of the ...
ReverseFlowControl's user avatar
1 vote
0 answers
40 views

What is the form of the incomplete Eisenstein series on PGL_2(C)?

Let $F$ be an imaginary quadratic number field. Let $G = \mathrm{PGL}_2(\mathbb{C}) $ and $\varGamma = \mathrm{PSL}_2(\mathcal{O}_F)$. We have the Iwasawa decomposition $G = NAK$ where $K = \mathrm{...
Misaka 16559's user avatar
2 votes
0 answers
190 views

Zeros of the semiprimes

Let $P$ be the prime zeta function $$ P (s) = \sum_{p\, \in\text{ primes}} \frac 1 {p^s} = \frac {1} {2^s} + \frac {1} {3^s} + \frac 1 {5^s} + \frac 1 {7^s} + \frac 1 {11^s} + \cdots $$ and define the ...
martin's user avatar
  • 1,893
13 votes
0 answers
304 views

Upper bound on prime powers in interval

I just spent a full day on the brutish and thankless task of proving that the Brun-Titchmarsh bound holds for prime powers (including primes), and not just for primes, in the following senses: (a) the ...
H A Helfgott's user avatar
  • 19.3k
2 votes
0 answers
66 views

The logarithmic derivative of a twisted L-function?

Let $F$ be a quadratic number field with class number $h_F = 1$. Let $\zeta_F$ be the Dedekind zeta function, we have $$ \frac{\zeta_F ' (1+it)}{\zeta_F (1+it)} \ll \frac{\log t}{\log\log t} .$$ (I ...
Misaka 16559's user avatar
1 vote
0 answers
156 views

Integers with $k$ prime factors, in terms of the Möbius function

A $k$-free integer is an integer $n$ such that there is no $k$th power dividing $n$. It is well known (see Murty's Problems in Analytic Number Theory q1.18 for instance) that \begin{equation}\sum_{d^k|...
alidixon222's user avatar
5 votes
1 answer
209 views

A limit related to quasi-periodic function

Let us consider $V(x) = 2-\sin(x) - \sin(\sqrt{2} x)$ on $x\in \mathbb{R}$ so that $V(x)>0$ everywhere. One can see that $$ \frac{C_1}{t^2} \leq \min_{|x|\leq t} V(x)\leq \frac{C_2}{t^2} $$ ...
Sean's user avatar
  • 313
3 votes
1 answer
285 views

Does this condition characterise intervals, among subsets of the real line?

For a real number, $c\in \left]0,1\right[$, consider the following property $\mathbf(\mathbf P_c\mathbf)$ of subsets $A$ of $\mathbb R$: $\mathbf(\mathbf P_c\mathbf)$ For every bounded set $B\subset \...
Pietro Majer's user avatar
  • 56.3k
3 votes
1 answer
268 views

Where to find or how to prove that the ratio of two Bernoulli polynomials is increasing?

It is well known that the classical Bernoulli polynomials $B_j(t)$ are generated by \begin{equation*} \frac{s\operatorname{e}^{ts}}{\operatorname{e}^s-1}=\sum_{j=0}^{\infty}B_j(t)\frac{s^j}{j!}, \quad ...
qifeng618's user avatar
  • 828
0 votes
1 answer
235 views

Real part of the Riemann zeta function

Consider the real part of the Riemann zeta function on the critical line. Are there any results for the number of zeros of this real function in the interval [0,T]?
Autovetor's user avatar
2 votes
0 answers
144 views

Large prime divisors of values of a polynomial, in a given residue class

Let $f(X) \in \mathbb{Z}[X]$ be an irreducible polynomial of degree $d \geq 2$. Let $q \in \mathbb{N}$ be an integer, and let $q \mathbb{Z} + r$ be a residue class that contains infinitely many primes ...
Jakub Konieczny's user avatar
1 vote
0 answers
167 views

Upper bound on sum of Lambda(n) over short interval

I am looking for a bound of type $$\sum_{x<n\leq x+y} \Lambda(n) \leq \frac{\log(x+y)}{\log y} \cdot 2y$$ (or better). Of course such a bound has to exist: the idea of the proof of Brun-Titchmarsh (...
H A Helfgott's user avatar
  • 19.3k
6 votes
1 answer
303 views

A conjectured series expression for the Riemann $\xi$-function and/or Completed L-series. Could this be proven?

This post builds on an MSE question about a conjectured series expression for the Riemann $\xi$-function: $$\xi(s) = \xi(1-s) = \sum_{n=1}^\infty (-1)^{n+1}\,\big(\xi\left(s+in\right)+\xi\left(1-s+in\...
Agno's user avatar
  • 4,179

1
2 3 4 5
58